LSAT and Law School Admissions Forum

Get expert LSAT preparation and law school admissions advice from PowerScore Test Preparation.

 Administrator
PowerScore Staff
  • PowerScore Staff
  • Posts: 8919
  • Joined: Feb 02, 2011
|
#43381
Please post your questions below! Thank you!
 tomthomse@gmail.com
  • Posts: 5
  • Joined: May 15, 2020
|
#76036
Hi, could you please explain why D is correct? I tried to diagram this one, and the closest I could get to paralleling the stimulus was C. Thanks for any help.
 Christen Hammock
PowerScore Staff
  • PowerScore Staff
  • Posts: 61
  • Joined: May 14, 2020
|
#76125
Hi Tom!

Let's take this from the beginning. The flaw in this passage is Mistaken Reversal: we can't conclude that ending global warming requires economic incentives just because offering economic incentives would be sufficient to decrease current reliance on fossil fuels.

Answer Choice (D) makes exactly this mistake: just because increasing salaries would be sufficient to keep good teachers in the profession doesn't mean that this increase is necessary.

Answer Choice (C) doesn't actually include a flaw! It's a valid logical chain. If graduating from high school is necessary to go to college, and going to college is necessary to get a professional job, it's valid to skip the middle condition (going to college) and conclude that going to high school is necessary for getting a professional job!
 Shaela L. Hayes
  • Posts: 10
  • Joined: Jun 01, 2021
|
#88103
Hi!

Can you please tell me where I went wrong in my initial diagram? And thoroughly explain how to diagram "required if" statements?

Premise: (Halt global warming) --> (Lower reliance on fossil fuels)
Premise: (Economic incentives) --> (Lower reliance on fossil fuels)
Conclusion: (Economic incentives) --> (Halt global warming)

Thanks!
User avatar
 Poonam Agrawal
PowerScore Staff
  • PowerScore Staff
  • Posts: 71
  • Joined: Apr 23, 2021
|
#88136
Hi Shaela,

Your diagram was almost there! Remember that conditional statements take the form of Sufficient :arrow: Necessary. If something is required, it is the necessary condition.

Here is how we can diagram the stimulus:

DRF = decreased reliance on fossil fuels
GWH = global warming halted
EI = economic incentives

Sentence 1: GWH :arrow: DRF
Sentence 2: EI :arrow: DRF
Sentence 3: GWH :arrow: EI (it says economic incentives are required to halt global warming)
Answer choice (D) can be diagrammed as follows:

KGT = keeping good teachers
IE = improving education
ITS = increase in teachers' salaries

Sentence 1: IE :arrow: KGT
Sentence 2: ITS :arrow: KGT
Sentence 3: IE :arrow: ITS
See the similarity between answer choice (D) and the stimulus? That's how you know we have found the parallel flaw - hope that helps!
User avatar
 lawlandmem
  • Posts: 9
  • Joined: Mar 24, 2021
|
#89269
Why is B wrong?
User avatar
 atierney
PowerScore Staff
  • PowerScore Staff
  • Posts: 215
  • Joined: Jul 06, 2021
|
#89574
So, it's important to remember that what we are doing in each parallel flaw question is to parallel the reasoning and all of its components in the answer choice. Thus, any question that does not do this is incorrect.

For the stimulus's argument, we have the following conditional statements: if global warming is to be halted, one must decrease reliance on fossil fuels. If economic incentives to develop alternative energy sources were present, then global reliance would decrease. It then concludes that if one wishes to end global warming, one must develop alternative energy sources. The flaw here is that it confuses the sufficient condition in second conditional statement with the necessary condition in the first, thus attempting to construct a chain using conditional statements whose common feature is a necessary condition (chains are constructed eliminating the necessary condition in one with the sufficient condition in the other).

B is wrong because it does not follow the flawed reasoning in the stimulus. For B we have the following conditionals: if exercise daily, then you must have good health, if you have good health then must have happy life, if you have good health, then must exercise daily. The conclusion here is simply a reversal of the first conditional statement, a mistaken reversal, but that doesn't parallel the reasoning in the stimulus. We do not have the similar confusion of the sufficient condition in the second conditional statement with the necessary condition in the other.

Let me know if you have further questions on this.

Get the most out of your LSAT Prep Plus subscription.

Analyze and track your performance with our Testing and Analytics Package.